7

Let $(N_t)_{t \geq 0}$ be a Poisson process of intensity $1$, and for each $\lambda>0$ and $t \geq 0$ let $$ W^{(\lambda)}_t = \sqrt{\lambda} \int_0^t (-1)^{N_{\lambda s}} \, ds = \frac{1}{\sqrt{\lambda}} \int_0^{\lambda t} (-1)^{N_s} \, ds. $$

Is it the case that for each $T>0$, as $\lambda \to \infty$ the law of the $C([0,T],\mathbb{R})$-valued random variable $(W^{(\lambda)}_t)_{t \in [0,T]}$ converges weakly to the Wiener measure on $C([0,T],\mathbb{R})$?

(Here, $C([0,T],\mathbb{R})$ is equipped with the topology of uniform convergence.)

Remark. If the answer is yes, then this may be a more physically intuitive way of thinking about Wiener processes than as the limit of a simple random walk: a model of random back-and-forth collision times "feels more physically motivated" (particularly when trying to visualise physical Brownian motion of particles) than a random decision to move either left or right at every time-step of a seemingly arbitrary fixed duration. It would also be a good way of formalising the notion that ("unbounded") one-dimensional Gaussian white noise can be obtained as a limit of ("bounded") dichotomous Markov noise.

If the answer to the question is yes, then this seems like a very basic fact; are there any references with this fact (either as a theorem or an exercise)?


My very crude intuition for a positive answer:

It "feels obvious" that for large $\lambda$, the stochastic process $(W_t^{(\lambda)})_{t \geq 0}$ has "approximately" stationary independent increments, and $\mathbb{E}[W_t^{(\lambda)}] \approx 0$ for all $t \geq 0$.

So now, for fixed $\tau>0$, let us consider the shape and variance of the distribution of $W_\tau^{(\lambda)}$. For each $n \in \mathbb{N}$, let $T_n=\inf\{t>0:N_{\lambda t}=n\}$. Since the random variables $T_i-T_{i-1}$ are independent and exponentially distributed with variance $\frac{1}{\lambda^2}$, the variance of $W_{T_n}^{(\lambda)}$ is $\frac{n}{\lambda}$, and applying the central limit theorem to $T_1 + (T_3-T_2) + (T_5-T_4) + \ldots$ and to $(T_2-T_1) + (T_4-T_3) + (T_6-T_5) + \ldots$ gives that for large $n$, the distribution of $W_{T_n}^{(\lambda)}$ is approximately normal in shape. Hence the distribution of $Y:=W_{T_{\lfloor \lambda\tau \rfloor}}^{(\lambda)}$ is approximately a normal distribution with a variance of approximately $\tau$. Now if we fix a small $\varepsilon>0$, writing $I_\varepsilon$ for the stochastic interval $$ I_\varepsilon = [T_{\lfloor \lambda(\tau - \varepsilon) \rfloor},T_{\lfloor \lambda(\tau + \varepsilon) \rfloor}], $$ taking sufficiently large $\lambda$ should give that

  • $\tau \in I_\varepsilon$ with high probability, and
  • the random variable $\max_{t \in I_\varepsilon} |W_t^{(\lambda)}-Y|$ is close to $0$ with high probability.

Hence it seems intuitive that the law of $W_\tau^{(\lambda)}$ should be approximately equal to the law of $Y$, and so in particular, should be approximately normally distributed with approximate variance $\tau$.


I've now posted a related more general question at MathOverflow, https://mathoverflow.net/questions/360363. If the answer to that question is yes, then the answer to this one should be as well (by the argument of zhoraster using the generalised invariance principle asked about in that question).

  • The convergence to $W$ follows the same path, grouping the variables by twos, just instead of CLT use the invariance principle. – zhoraster May 11 '20 at 14:19
  • Sounds very much like the continuous time random walk: https://en.wikipedia.org/wiki/Continuous-time_random_walk – user619894 May 11 '20 at 14:59
  • @user619894 I don't think this is a continuous-time random walk. Part of the definition of a random walk (in continuous or discrete time) is usually some notion of "i.i.d. increments" (e.g. as in your linked Wikipedia article). By contrast, the process I'm describing alternates deterministically between moving left and moving right, only with random waiting times. – Julian Newman May 13 '20 at 15:53
  • @zhoraster Let $\Delta T_n$ be the $n$-th waiting time for the Poisson pocess $(N_{\lambda t})$. If I was looking at the process whose value at time $\frac{2n}{\lambda}$ is $\sqrt{\lambda}(\Delta T_1-\Delta T_2+\Delta T_3-\Delta T_4+\ldots-\Delta T_{2n})$, then I can see that the I.P. would give the desired result. But the whole point is that I'm looking at the process whose value is $\sqrt{\lambda}(\Delta T_1-\Delta T_2+\Delta T_3-\Delta T_4+\ldots-\Delta T_{2n})$ not at time $\frac{2n}{\lambda}$ but rather at time $\Delta T_1+\Delta T_2+\Delta T_3+\Delta T_4+\ldots+\Delta T_{2n}$. – Julian Newman May 13 '20 at 16:02
  • You're right. But, thanks to the SLLN, $\Delta T_1+\Delta T_2+\Delta T_3+\Delta T_4+\ldots+\Delta T_{2n} \sim \frac{2n}{\lambda}$, so this does not matter at all. – zhoraster May 14 '20 at 07:52
  • @zhoraster After seeing your original comment, I was also thinking that some argument along these lines might overcome the problem - but I'm pretty sure we need more than just LLN. If we had something like $\sqrt{\lambda} \max {1 \leq n \leq \lfloor \frac{2}{\lambda} \rfloor} |\Delta T_1 + \ldots + \Delta T{2n}-\frac{2n}{\lambda}| \to 0$ in distribution as $\lambda \to \infty$, then this would be enough - but unfortunately, I'm pretty sure CLT tells us that we don't! – Julian Newman May 14 '20 at 14:01
  • You wrote something weird: when $\lambda >2$, you have $\max_{1\le n \le 0}$. – zhoraster May 14 '20 at 15:03
  • @zhoraster Sorry, the subscript is meant to say $1 \leq n \leq \lfloor \frac{\lambda}{2} \rfloor$. (Actually, I could replace $\frac{\lambda}{2}$ with $c\lambda$ for any constant $c$. If we are thinking about convergence on $[0,1]$, then we basically want $c=\frac{1}{2}$.) – Julian Newman May 14 '20 at 15:11
  • And why do you need $\max$ for the invariance principle? – zhoraster May 14 '20 at 15:20
  • @zhoraster On SE's automated recommendation, I've now moved the discussion to "chat" (see the comment above for the link). – Julian Newman May 14 '20 at 15:36
  • @zhoraster Thanks for your help, on the basis of which I've now written a complete answer to the question. – Julian Newman Jun 02 '20 at 12:09

1 Answers1

4

With the help of MathOverflow, at https://mathoverflow.net/questions/360363/, I can now turn zhoraster's suggestion into a complete answer:

Define the stochastic process $(\tilde{W}_t^{(\lambda)})_{t \geq 0}$ such that $\tilde{W}_t^{(\lambda)}$ agrees with $W_t^{(\lambda)}$ at every second event of the Poisson process $(N_{\lambda t})_{t \geq 0}$ and is linearly interpolated in between. That is to say, $$ \tilde{W}_{\frac{1}{\lambda}(rS_{2n} \, + \, (1-r)S_{2n+2})}^{(\lambda)} \ = \ rW_{\frac{1}{\lambda} S_{2n}}^{(\lambda)} \ + \ (1-r)W_{\frac{1}{\lambda} S_{2n+2}}^{(\lambda)} \quad \textrm{for all } n \geq 0, \, r \in [0,1] $$ where $$ S_n \, := \, \inf\{t \geq 0 : N_t = n\}. $$ Let $D_n=S_{n+1}-S_n$. We can apply the generalised Donsker invariance principle in the linked MO question, with $\Delta_n=D_{2n}+D_{2n+1}$ and $X_n=D_{2n}-D_{2n+1}$, to yield that on any compact time-interval the process $\tilde{W}_t^{(\lambda)}$ converges in distribution (w.r.t. the topology of uniform convergence) to the Wiener process.

So it only remains to control the difference $\tilde{W}_t^{(\lambda)} - W_t^{(\lambda)}$. For any $T>0$, $$ \max_{t \in [0,T]} |\tilde{W}_t^{(\lambda)} - W_t^{(\lambda)}| \ \leq \ \frac{\max\{ D_n : 0 \leq n \leq N_{\lambda T}+1 \}}{\sqrt{\lambda}}. $$ Now for any $\varepsilon>0$, taking sufficiently large $\lambda$ will give that $\mathbb{P}(N_{\lambda T}+1 \leq \lceil 2\lambda T \rceil)>1-\frac{\varepsilon}{2}$; and since $$ \max\{ D_n : 0 \leq n \leq \lceil 2\lambda T \rceil \} - \log(\lceil 2\lambda T \rceil) $$ is convergent in distribution as $\lambda \to \infty$ (to the Gumbel distribution) and $\frac{\log(x)}{\sqrt{x}} \to 0$ as $x \to \infty$, it follows that for sufficiently large $\lambda$, $$ \mathbb{P}\left( \frac{\max\{ D_n : 0 \leq n \leq \lceil 2\lambda T \rceil \}}{\sqrt{\lambda}} > \varepsilon \right) \ < \ \tfrac{\varepsilon}{2} $$ and therefore $$ \mathbb{P}\left( \frac{\max\{ D_n : 0 \leq n \leq N_{\lambda T}+1 \}}{\sqrt{\lambda}} > \varepsilon \right) \ < \ \varepsilon. $$ Hence $\max_{t \in [0,T]} |\tilde{W}_t^{(\lambda)} - W_t^{(\lambda)}|$ converges in probability to $0$, and so it follows that $(W_t^{(\lambda)})_{t \in [0,T]}$ converges in distribution to the Wiener process on $[0,T]$.

  • For the convergence to the Gumbel distribution: https://math.stackexchange.com/questions/3519031/convergence-in-distribution-of-maximum-of-exponentially-distributed-random-varia/ – No-one Sep 26 '24 at 14:26